CS 281 Solutions to Test II

June 3, 1997

1 (a) tex2html_wrap_inline91
(b) tex2html_wrap_inline93

2 (a) tex2html_wrap_inline95
(b) tex2html_wrap_inline97

3 (a) No. For example let tex2html_wrap_inline99 and tex2html_wrap_inline101 . Then tex2html_wrap_inline103 .
(b) Take an arbitrary element x in the universe. Then
tex2html_wrap_inline107
tex2html_wrap_inline109 .

4 (a) Let ODD denote the set of odd integers.
Basis Clause: tex2html_wrap_inline113
Inductive Clause: If tex2html_wrap_inline115 , then tex2html_wrap_inline117 and tex2html_wrap_inline119 .
Extremal Clause: Nothing is in ODD unless it is obtained from the Basis and Inductive Clauses.

(b) Let ND3 denote the desired set.
Basis Clause: tex2html_wrap_inline125
Inductive Clause: If tex2html_wrap_inline127 , then tex2html_wrap_inline129 .
Extremal Clause: Nothing is in ND3 unless it is obtained from the Basis and Inductive Clauses.

5 (a) Basis Step: Let n = 0. Then LHS = 3*0 = 0, and RHS = 3*0*(0+1)/2 = 0. Hence LHS = RHS.
Inductive Step: tex2html_wrap_inline141
Since tex2html_wrap_inline143 by the induction hypothesis,
tex2html_wrap_inline145 .

(b) Basis Step: Let n = 1. Then tex2html_wrap_inline149 and tex2html_wrap_inline151 . Since tex2html_wrap_inline153 by the hypothesis, tex2html_wrap_inline155 .
Inductive Step: Assume that tex2html_wrap_inline157 holds and prove that it holds for k+1.
Take an arbitrary element tex2html_wrap_inline161 . We try to show that tex2html_wrap_inline163 . tex2html_wrap_inline165 by the definition of tex2html_wrap_inline167 .
Hence tex2html_wrap_inline169 .
By the induction hypothesis if tex2html_wrap_inline171 , then tex2html_wrap_inline173 .
Also if tex2html_wrap_inline175 then tex2html_wrap_inline177 .
Hence tex2html_wrap_inline179 . Hence tex2html_wrap_inline181 .
Hence tex2html_wrap_inline163 .





S. Toida
Wed Jun 18 11:07:28 EDT 1997